¿Dos voltímetros conectados a los mismos terminales pueden mostrar valores diferentes? Circuito con FEM inducida

El circuito que se muestra a continuación consta de dos resistencias conectadas en serie que forman un circuito cerrado. No hay fuentes eléctricas en ese bucle. Supongamos que la resistencia de los cables que conectan las resistencias y forman el bucle es cero. Dentro del bucle hay un campo magnético externo cambiante que induce dentro de ese bucle una FEM constante de 1 V.

ingrese la descripción de la imagen aquí

ingrese la descripción de la imagen aquí

La corriente dentro del circuito se calcula usando la Ley de Ohm:

ingrese la descripción de la imagen aquí

El problema fue presentado por el ex profesor del MIT Walter Lewin. El problema se presentó en este video (a partir del minuto 35 del video): 8.02x - Lect 16 - Inducción electromagnética, Ley de Faraday, Ley de Lenz, SUPER DEMO

En el video se muestra que dos voltímetros conectados a los mismos terminales muestran valores diferentes.

¿Dos voltímetros conectados a los mismos terminales pueden mostrar valores diferentes?

Mi solución a este problema sería:

Para medir el voltaje entre los puntos D y A, el circuito debe dividirse en dos mitades, la mitad izquierda del bucle y la mitad derecha del bucle. Si se considera que la FEM inducida se distribuye por igual a través del bucle, en cada mitad del bucle hay una FEM inducida de 0,5 V:

ingrese la descripción de la imagen aquí

La polaridad de los campos electromagnéticos inducidos está de acuerdo con la ley de Lenz.

ingrese la descripción de la imagen aquí

El voltaje entre los puntos D y A al escribir la ecuación a través de la mitad izquierda del bucle es:

ingrese la descripción de la imagen aquí

El voltaje entre los puntos D y A al escribir la ecuación a través de la mitad derecha del bucle es:

ingrese la descripción de la imagen aquí

La conclusión es que existe un valor de voltaje único entre los puntos D y A, y el valor medido debe ser de 0,4 V.

Ahora vamos a conectar al circuito entre los puntos D y A dos voltímetros. Los dos voltímetros conectados junto con los cables que los conectan al circuito principal forman un segundo bucle. Supongamos que la resistencia del cable que conecta los voltímetros y forma el segundo bucle es cero. La mitad izquierda del segundo bucle está conectada al voltímetro entre los puntos C y B, la mitad derecha del segundo bucle está conectada al voltímetro entre los puntos F y E. También hay una FEM inducida de 1V dentro del segundo bucle. Si se considera que la FEM inducida se distribuye por igual a través del segundo bucle, el circuito final se puede representar con el modelo que se muestra a continuación.

ingrese la descripción de la imagen aquí

El voltaje entre los puntos C y B se define con las siguientes ecuaciones:

ingrese la descripción de la imagen aquí

El voltaje entre los puntos F y E se define con las siguientes ecuaciones:

ingrese la descripción de la imagen aquí

Los cálculos anteriores coinciden con la medición realizada por el profesor Walter Lewin.

Mi pregunta principal es: ¿ Cuál es la diferencia de potencial teórico entre los puntos D y A?

Debido a los campos eléctricos y magnéticos cambiantes, hay corriente portadora de carga y corriente de desplazamiento. En su cálculo, se perdió la contribución actual de desplazamiento.
@VVT no hay cambio de campo eléctrico.
¡¡¡Impresionante!!! ¡Profesor Lewin gobierna!
Si tenía un voltímetro conectado y se colocó sobre el campo B cambiante de modo que los cables que lo conectan a D y A no recibieran voltaje inducido, el valor medido podría ser de 0,4 voltios.
Básicamente, tiene razón aquí, los cables del medidor obtienen un voltaje inducido del campo cambiante y, por lo tanto, puede sumar los voltajes y obtener el resultado esperado del Kirchhoff estándar ...
Walter lewin ha estado impulsando esta idea de campos no conservadores, incapacidad para modelar este escenario... Etc... durante años, y básicamente ha demostrado una cosa: simplemente no lo entiende correctamente. El canal Electro Boom volvió a hacer sus experimentos con el sondeo adecuado y mostró que Lewin simplemente no lo entiende, pero debido a que el tipo parece un "científico" y proviene del MIT, pocas personas están dispuestas a llamarlo.
@Andy, también conocido como HAY campo eléctrico y varía en el espacio. En esta configuración resulta que el campo eléctrico tiene circulación y por lo tanto no es conservativo. En otras palabras, E no depende del tiempo, pero sí el potencial escalar y el potencial vectorial. Un voltímetro lee el componente relacionado con el potencial escalar de E, no uno que contiene rotaciones. Consulte physicsforums.com/insights/… , physics.princeton.edu/~mcdonald/examples/lewin.pdf .
Estoy de acuerdo con @David Molony en su crítica del experimento disruptivo. Solo agrego que el campo no conservador no conduce a perpetuum mobile.
"Creo que todo este argumento es más una lección de sociología que de física": Paul Rako en electronicdesign.com/technologies/analog/article/21808474/… . Campos no conservativos para 'condición de betatrón', parámetros agrupados para 'voltímetro en una región de conexión múltiple'.
Para responder a la pregunta del título: Sí, si uno o ambos están mal calibrados.
@DavidMolony Lewin siempre ha tenido razón, y Electroboom está malinterpretando a Feynman. Este efecto es solo una consecuencia elemental de tener un campo no conservativo, algo que muchos ingenieros no están equipados para manejar, al parecer. Aquí no hay error de medición. Es simplemente física básica.
Su circuito teórico es imposible. No tiene ningún componente en el que se pueda inducir un voltaje. Sin fuente de voltaje, sin corriente, sin voltaje a través de las resistencias. "En el video se muestra que dos voltímetros conectados a los mismos terminales muestran valores diferentes". - Eso es imposible, por definición.
@BruceAbbott, ese es el objetivo de esta pregunta, establecer si los cálculos son incorrectos o si el modelo de circuito está incompleto.
Lo que realmente está preguntando es si un dispositivo físico puede hacer que dos voltímetros muestren lecturas diferentes cuando se conectan a los mismos terminales. Obviamente , esto es posible, pero no se puede explicar haciendo referencia al circuito que se muestra, que no representa la construcción física del sistema.
@BruceAbbott la pregunta está relacionada con el video. La pregunta es si hay un voltaje definido o no definido entre los puntos D y A
Desafortunadamente, mi computadora no puede reproducir videos de YouTube y, de todos modos, no estoy dispuesto a verlos. ¿Puede insertar un cuadro del video que muestre la configuración física (incluido todo el cableado, etc.)?
Vídeo muy impresionante. Pero, ¿qué pasa con la pizarra en 36.15 '?

Respuestas (4)

, dos voltímetros conectados a la misma pareja de terminales pueden mostrar simultáneamente dos valores diferentes. La razón es que, dado que la ruta del circuito encierra una región de flujo magnético variable, la 'diferencia de potencial teórica' entre el punto D y A no está definida (únicamente).

En realidad, eso es todo lo que hay que hacer .

Porque si su circuito se encuentra en una región de campo magnético variable, entonces todos los caminos entre nodos se encuentran en una región dB/dt, y dado que la integral de línea del campo eléctrico a lo largo de cualquier camino cerrado no siempre será cero (esto es física básica) , no es posible definir una función potencial tal que la integral de línea de E de D a A solo dependa de los valores en D y A . Dependerá del camino. (Esto es cálculo básico)

Dado que esta noción suele encontrar cierta resistencia para ser aceptada, vale la pena profundizar en su origen y qué implica en detalle. Responder a todas las dudas que surgieron de las numerosas interacciones en internet luego de que Walter Lewin hiciera accesible a las masas su 'superdemo', requiere mucho espacio. Para la gente de TL;DR, el resto de esta publicación está estructurada de esta manera:

  1. Una definición de diferencia de potencial
    Donde definimos lo que generalmente se entiende por voltaje en la teoría de circuitos y por qué puede reducirlo a una diferencia de potencial (pista: la irrotacionalidad del campo eléctrico hace que el voltaje sea una función solo de los puntos finales)
  2. Teoría de circuitos concentrados I: Reglas de Kirchhoff
    Donde vemos qué condición física hace posible tener un campo E irrotacional y hacer uso del concepto de voltaje como diferencia de potencial. (Pista: es la ausencia de un flujo magnético variable: la ley de voltaje de Kirchhoff es solo la ley de Faraday cuando dphi/dt = 0)
  3. Teoría de circuitos agrupados II: Regiones limitadas de no conservación fuera del camino del circuito
    Donde tratamos de salvar KVL ocultando regiones circunscritas de flujo magnético variable dentro de los componentes magnéticos (nota: esto es lo que Lewin se refiere como la física apestosa de 'tomar el derecho lado de la mano en el lado izquierdo' y lo que llamo 'nuevo KVL' o 'KVL enmendado': si la ruta del circuito no incluye un campo magnético variable, podemos pretender que hay un voltaje único expresable como diferencia de potencial simplemente sin mirar dentro los componentes magnéticos)
  4. El anillo de Romer-Lewin, o: el circuito no agrupable
    Donde vemos que cuando la ruta del circuito encierra un flujo magnético variable KVL ya no es recuperable, y el voltaje en su circuito se vuelve dependiente de la ruta (lo que significa: ya no es posible expresarlo como diferencia de potencial)
  5. Potenciales escalares y vectoriales
    Donde echamos un vistazo a una definición alternativa de voltaje que admite un potencial escalar. (pista: esto es lo que yo llamo la maniobra de McDonald, y consiste en separar el campo eléctrico total en dos partes constituyentes, una conservadora que es causada por la interacción coulombiana, una rotacional que es causada por la inducción magnética)
  6. El papel de las cargas superficiales
    Donde observamos lo que hace que el campo total en el conductor y las resistencias sea como es.
  7. El concepto de inductancia parcial
    Donde observamos cómo tratar de modelar el voltaje inducido con bobinas 'secundarias' agrupadas parciales corresponde a una situación física diferente y cómo las inductancias parciales pueden usarse (con la debida precaución) como una herramienta en el campo de Compatibilidad electromagnética

.

1 - Una definición de diferencia de potencial

Cuando defines el voltaje como la diferencia de potencial entre dos puntos

VBA = VB - VA = Vab (pasar de a a b)

está asumiendo implícitamente que, además de una constante aditiva, la función potencial está determinada de manera única, es decir, la integral de trayectoria (negativa de) del campo eléctrico de a a b

voltaje como energía por unidad de carga

no depende de la ruta de integración particular elegida integración. Esta situación requiere que el campo eléctrico E sea conservativo o, y este nombre es revelador, irrotacional .

Si el campo eléctrico se hace rotacional, como sucede en presencia de un campo magnético variable, entonces puede obtener diferentes resultados a la vez para Vab, dependiendo de la ruta particular que elija para calcular la integral: sigue la ruta que pasa a través de R1, obtienes 0.9V; vas por el camino que pasa por R2 obtienes -0.1V; vas por un camino arbitrario que sigue el perfil de Snoopy dentro y fuera del conductor y las resistencias, obtienes 0.741V, lo que sea. (Tenga en cuenta que no estoy mencionando sondas en absoluto). Los puntos finales por sí solos ya no son suficientes para determinar el voltaje de manera única.

Aquí está la desambiguación entre las dos convenciones que voy a usar. Dado que sería demasiado usar una letra diferente para denotar voltaje dependiente e independiente de la ruta, usaré V para ambos, dejando que el texto pequeño los diferencie. Entonces, Vab significa

notación para el voltaje dependiente de la ruta

y es la energía por unidad de carga requerida para ir del punto A al punto B a lo largo del camino gamma(a->b). Denotaré este voltaje, generalmente dependiente de la ruta , con Vab, con letras minúsculas desde el punto de inicio hasta el punto final. Cuando el campo eléctrico es irrotacional, existe una función de potencial V tal que

existe una función potencial

de modo que la integral de línea se vuelve independiente de la ruta y solo se deben especificar los puntos finales A y B, no la ruta gamma que los une (cualquier ruta servirá).

El voltaje independiente del camino expresa la diferencia de potencial.

Llamaré a este voltaje 'diferencia de potencial' y lo denotaré con VBA, con letras mayúsculas ordenadas desde el punto final hasta el punto inicial. Nota: el punto final es lo primero, ya que adopto la convención VBA = VB - VA (representada gráficamente con una flecha que va de A a B). Esto ayudará aún más a diferenciar entre los dos.

Una pequeña digresión
Realmente no hay nada extraño en tener integrales de línea dependientes del camino, de hecho son campos conservativos que son casi mágicamente especiales. En el mundo real, es normal considerar que una función calculada por integración a lo largo de un camino depende del camino particular en sí y no solo de sus extremos.

Considera la energía gastada para llegar a la cima de una montaña en tu auto desde el mismo estacionamiento y midámosla en galones de gasolina: si eliges un camino recto necesitas 10 galones, si eliges una espiral suelta necesitas 20 galones, una espiral suelta contra el viento y son 25 galones, pero si es junto con el viento son 18 galones; un camino tambaleante con ida y vuelta y son 35 galones. (Es una montaña muy alta, ojo). ¿Le resultaría extraño que la respuesta a "¿cuál es el consumo teórico de gasolina para ir de A a B?" era "depende"? Por supuesto que no, te sorprenderías de lo contrario. (Y, sin embargo, podría pensar en un consumo mínimo que podría considerarse 'único', volveré a eso más adelante, cuando presente el vector potencial).

Además, ¿esperaría que su tanque de gasolina se llene simplemente bajando al estacionamiento? Creo que te sorprenderías mucho si eso sucediera. Y, sin embargo, eso es lo que sucede con los campos conservadores: completar el círculo te dará una integral de línea cero. Son las cosas contrarias a la intuición, no al revés.

2 - Teoría de circuitos agrupados I: Kirchhoff rulez

La teoría de circuitos concentrados es tan fácil porque se basa en este tipo de campos especiales, casi mágicos. La teoría es tan simple y hermosa que muchos ingenieros no pueden dejarla de lado y tratar de usarla incluso cuando no es aplicable. La suposición fundamental de la teoría de circuitos agrupados es que debe poder definir el voltaje entre dos puntos independientemente de la ruta. Considerando la definición de voltaje dada arriba, esto significa que

diferencia de potencial

y en particular, si elegimos un camino de integración cerrado, es decir a=b, la circulación de E debe ser cero

circulación en campos irrotacionales

Resulta que tener circulación cero para el campo eléctrico a lo largo de cualquier camino cerrado es una condición necesaria y suficiente para la existencia de una función potencial. Ahora, dado que una de las leyes fundamentales de la física, la ley de Faraday, establece que

Forma integral de la Ley de Faraday

se deduce que si queremos poder expresar el voltaje como una diferencia de potencial independiente del camino, debemos evitar las regiones de cambio de flujo magnético. Feynman es muy claro al respecto en sus Lectures: Volume II, Ch. 22, "Circuitos de CA" (énfasis mío).

Suponga que tenemos un circuito que consta de un generador y varias impedancias conectadas entre sí, como se muestra en la figura 22-9. De acuerdo con nuestras aproximaciones, no hay campo magnético en la región fuera de los elementos individuales del circuito .

ingrese la descripción de la imagen aquí

Por lo tanto, la integral de línea de E alrededor de cualquier curva que no pase por ninguno de los elementos es cero. Considere entonces la curva Γ que muestra la línea discontinua que da la vuelta al circuito en la figura 22-9. La integral de línea de E alrededor de esta curva se compone de varias partes. Cada pieza es la integral de línea desde un terminal de un elemento de circuito al otro. A esta integral de línea la hemos llamado caída de tensión en el elemento del circuito. La integral de línea completa es entonces solo la suma de las caídas de voltaje en todos los elementos del circuito:

ingrese la descripción de la imagen aquí

Como la integral de línea es cero, tenemos que la suma de las diferencias de potencial alrededor de un bucle completo de un circuito es igual a cero:

ingrese la descripción de la imagen aquí

Este resultado se deriva de una de las ecuaciones de Maxwell: que en una región donde no hay campos magnéticos, la integral de línea de E alrededor de cualquier bucle completo es cero.

Entonces, KVL es solo un caso especial particular de la ley de Faraday, cuando el campo eléctrico se comporta de esa manera mágica que recargará su tanque de gasolina cuando regrese de la cima de la montaña al estacionamiento. Si la ruta de su circuito encierra un campo magnético variable, entonces no tiene suerte: los voltajes en su circuito no tendrán valores únicos para los mismos puntos finales, sino que también dependerán de la ruta particular a lo largo de la cual calcule la integral de línea. Pero observe que Feynman dice:

"Según nuestras aproximaciones, no hay campo magnético en la región fuera de los elementos individuales del circuito ".

Entonces, parece que hay espacio para alguna locura de campo magnético. Bueno, dentro de lo razonable.

3 - Teoría de circuitos agrupados II: Regiones limitadas de no conservación fuera del camino del circuito

Si tuviéramos que evitar por completo las regiones de campo magnético (y de desplazamiento) variable, nos quedaríamos atrapados con circuitos simplemente resistivos. Veamos si podemos encontrar una solución que permita elementos dinámicos como inductores (y capacitores) en la imagen sin renunciar a las herramientas simples representadas por KVL (y KCL). Resulta que podemos, si agregamos la condición adicional de que toda la locura del campo variable se deje fuera de la ruta del circuito confinándola dentro de los componentes agrupados.

Dejar la región magnética variable fuera de la ruta del circuito nos permitirá definir voltajes en el circuito de una manera única y usar las leyes de Kirchhoff para calcular voltajes y corrientes como lo hicimos con los circuitos resistivos. Al ocultar la complejidad de los campos magnéticos (y eléctricos) variables dentro de los componentes dinámicos, podemos pretender que el voltaje a través de ellos y la corriente a través de ellos todavía obedecen las reglas de Kirchhoff, mientras que, de hecho, en el fondo, están determinados por las leyes más generales. de Faraday-Neumann y Ampere-Maxwell.

Consideremos el siguiente circuito, con un generador, una resistencia, un inductor y un capacitor a lo largo del camino que conecta todas sus terminales:

Circuito agrupado con ruta de circuito segura

Las regiones de campo magnético variable se indican mediante regiones sombreadas y están relegadas dentro del elemento magnético. El camino cerrado se puede descomponer en los siguientes segmentos:

la ruta de circuito segura no encierra un campo magnético cambiante

Ahora, si, como sucede en esta imagen, la trayectoria del circuito no encierra una región de campo magnético variable, podemos escribir la integral de trayectoria del campo eléctrico a través del bucle cerrado abcda y estar seguros de que es cero. Esta es la condición requerida para considerar la función de voltajes de los puntos finales solamente, y para que KVL funcione (la suma de los voltajes a lo largo de un circuito cerrado es cero). KVL funciona porque, al no encerrar ningún campo magnético variable en la ruta de nuestro circuito Gamma, todos los voltajes pueden considerarse independientes de la ruta y no importa cómo integremos de una terminal a la otra.

Pero, ¿todos los voltajes en este circuito son realmente independientes de la ruta? Bueno, no . Simplemente pretendemos que lo son haciendo la vista gorda a los que no lo son.

Resulta que cuando nos encontramos con el componente magnético, podemos elegir caminos que van de un terminal al otro entrando o rodeando la región de flujo magnético variable. El voltaje calculado como una integral de línea a lo largo de esos caminos, en general, dependerá del camino.

diferentes caminos que unen las terminales - no todos son seguros No todos los caminos que van de un terminal a otro dan el mismo resultado, si vamos dentro del componente

Podemos tener caminos de C a D que crucen la región de campo magnético variable, de modo que haya bucles cerrados que pasen de C y D cuya integral de línea no sea cero. Esto significa que podemos tener múltiples valores para el voltaje entre C y D. En particular, hay diferentes valores para la integral de línea que va del punto C al punto D en el espacio entre la terminal (digamos un voltaje V) y otro valor diferente para la integral de línea a lo largo del camino que sigue la bobina dentro del conductor (y ese valor es cero).

Sin embargo, hay esperanza. Entre la infinidad de caminos a elegir, aquellos que no van dentro o alrededor de la región prohibida (como los indicados por γ 1 , γ 2 , y γ 3 ) todos dan el mismo resultado (de hecho, al elegirlos siempre terminamos con una ruta de circuito general que no encierra la región variable magnética).

Entonces, si cuando consideramos una ruta que conecta las terminales somos lo suficientemente inteligentes como para mantenernos alejados de la región de flujo magnético variable, nuestra ruta de circuito estará despejada y, si podemos engañarnos a nosotros mismos de que el voltaje de la bobina solo depende de los puntos finales (¡no es así!) - podremos usar una versión simulada de KVL (de la misma manera que saltando a los terminales del capacitor, podemos usar una versión simulada de KCL)

apesta

Ahora, el problema es encontrar cuál es ese valor de la integral de camino a lo largo de cualquier camino 'seguro' que va de una terminal a otra sin entrar o rodear la zona prohibida del elemento magnético. Resulta que podemos calcularlo fácilmente usando, lo adivinaste, la ley de Faraday.

La falsa 'diferencia de potencial' a través de una bobina

Vamos a considerar esta parte del circuito donde tenemos el valor de B ( t ) o Φ ( t ) está determinado por los efectos de una explosión nuclear, un imán que cae, otra bobina o incluso magia élfica (pero tenga en cuenta que no estoy considerando el caso de una bobina en movimiento, lo que significa que estamos en un marco de referencia donde la bobina CD está en reposo )

el elemento magnetico

Estamos interesados ​​en la integral de línea a lo largo de uno de los caminos 'seguros' de C a D, que llamaremos γ t mi r metro . Podemos calcularlo fácilmente considerándolo como parte de un circuito cerrado que rodea la región de flujo variable al atravesar el cobre (de modo que el campo eléctrico tenga un valor y una dirección conocidos o fáciles de calcular)

imagen

Podemos dividir la integral de línea en bucle cerrado en sus partes constituyentes, es decir, el camino que conecta los terminales de C a D ( γ t mi r metro ), y el camino que atraviesa el cobre de D a C ( γ C o pag pag mi r ):

imagen

lo que nos interesa es la primera integral del lado derecho, que corresponde a la 'diferencia de potencial' simulada en los terminales de la bobina. Podemos encontrarlo de inmediato si consideramos que la circulación del campo eléctrico a través del circuito cerrado tiene un valor dado por la ley de Faraday, y que la integral de línea que atraviesa el cobre es cero porque el campo eléctrico dentro de un conductor perfecto estacionario, incluso bajo la efecto de inducción - es cero:

imagen

y aquí está el resultado de nuestra 'diferencia de potencial' de maqueta:

ingrese la descripción de la imagen aquí

Pongo "VDC" entre comillas porque no es una diferencia de potencial real, ya que en general este voltaje depende del camino y no solo de los extremos C y D. Pero si podemos asegurarnos que lo referimos a caminos que no entrar o rodear las tripas del inductor, podemos fingir que tiene un solo valor.
El voltaje que ve en los terminales del inductor es la derivada temporal del flujo magnético phi. El signo del voltaje depende de la orientación del campo magnético y de cómo está cambiando (disminuyendo, aumentando) Si quieres, agrega eso Φ = L i (y en ese caso, dado que el flujo magnético es producido por la corriente en el bucle mismo, el signo del voltaje se seguirá de la ley de Lenz) o Φ = METRO i (aquí el signo también depende de cómo orientes las bobinas) y obtienes las conocidas relaciones para la inductancia propia y mutua. Pero este resultado también es válido para cambios en el campo magnético causados ​​por la caída de un imán o una explosión nuclear.

El campo cero en el conductor significa que no se acumula voltaje dentro de la bobina

Ahora bien, lo que más nos interesa es que, a pesar del voltaje que se presenta en los terminales, el campo eléctrico (total) dentro del conductor de cobre del que está hecha la bobina es cero. No hay 'acumulación de voltaje incremental' por así decirlo. Todo el voltaje aparece en los terminales. Así lo expresan Ramo, Whinnery y VanDuzer (p. 171 en la 2da edición, énfasis mío)

"El voltaje en las terminales del elemento magnético proviene de la tasa de cambio del flujo magnético dentro del inductor, que se muestra en la figura como bobina. Suponiendo primero que la resistencia del conductor de la bobina es insignificante, tomemos una línea cerrada integral del campo eléctrico a lo largo del conductor de la bobina, volviendo por el camino a través de los terminales, figura 4.2b. Como la contribución a lo largo de la parte del camino que sigue al conductor es cero , todo el voltaje aparece a través de los terminales".

¿Cómo es posible que la integral de trayectoria del campo eléctrico E (total, ver expansión posterior) sea cero dentro del conductor de la bobina, cuando podemos medir un voltaje en sus terminales? Bueno, al igual que en el caso de la inducción electrostática, la carga que produce el campo fuera del cobre es la misma carga responsable de convertirlo en cero dentro del cobre. En el caso electrostático la integral de trayectoria es cero tanto dentro como fuera del conductor; en el caso cuasiestático de inducción en un conductor estacionario, la integral de trayectoria es cero en el interior pero distinta de cero en el exterior.

dos inducciones

Si calcula la integral de línea del campo eléctrico que salta a lo largo de un camino seguro de un terminal al otro, obtiene valores distintos de cero (correspondientes al voltaje que es dado por la derivada del tiempo del flujo magnético) porque hay un total distinto de cero. campo eléctrico conservativo en el espacio entre los terminales, pero si calcula la integral de línea desde los mismos dos extremos yendo dentro del conductor, obtiene cero porque dentro del conductor perfecto en estas condiciones cuasiestáticas no hay campo eléctrico.

Entonces, ahí lo tiene: valores múltiples para el voltaje entre los mismos dos puntos en todos sus circuitos usando cualquier forma de inductor.

El truco que usamos en la teoría de circuitos es solo fingir que no vemos los infinitos valores que puede tener la integral de línea si elegimos caminos dentro de la región prohibida del componente, y solo miramos los buenos caminos fuera de ella que consistentemente dan el mismo valor. Al olvidar que es solo un valor entre muchos, llamamos a ese voltaje una 'diferencia de potencial' incluso si no hay una función de potencial .

Ahora, si tuviera que realizar mediciones de voltaje en su circuito, obtendría resultados consistentes, sin importar el camino que elija, siempre y cuando permanezca fuera y no rodee la región prohibida dentro del componente magnético . Es con esta condición que podemos pretender que todavía podemos hablar de diferencias de potencial y voltajes independientes de la trayectoria. El funcionamiento de su circuito se basa en ese voltaje 'único' ilusorio en las terminales del inductor. Pero si coloca sus sondas alrededor o dentro del componente magnético, cometerá un error de sondeo, porque jugar con la región prohibida destruirá esa ilusión.

Avance preliminar: ¿Por qué no puede funcionar el 'KVL modificado' en el ring Romer-Lewin?

El anillo de Romer-Lewin tiene algunos puntos en común con el circuito agrupado anterior, pero también algunas diferencias sorprendentes. De manera similar al circuito con un inductor, no hay un campo eléctrico total dentro del conductor, y todo el campo E termina entre las terminales de las resistencias (o componentes no magnéticos). La diferencia realmente grande es que ahora es la ruta del circuito la que rodea una región de flujo magnético variable, por lo tanto, siempre estamos dentro de la "región prohibida" donde KVL no funciona y no podemos "quedarnos fuera" como lo hicimos en el caso de el componente magnético aislado. Dado que, en general, la integral de línea del campo eléctrico total a lo largo de cualquier bucle cerrado no será cero sino igual a menos la derivada temporal del flujo magnético encerrado por la trayectoria,En general, los voltajes en el circuito dependerán de la ruta y debemos esperar múltiples valores para el voltaje entre dos puntos.

Esta multivaloración del voltaje no es el resultado de un error de medición, es solo una consecuencia del hecho de que todo el circuito se encuentra (rodea) la zona prohibida y, por lo tanto, KVL, ni siquiera el 'modificado', no puede funcionar allí. No puede sacar dos terminales y colocarlos en una ruta de circuito 'segura' que no rodee la zona prohibida, como hicimos con el inductor agrupado en el circuito anterior. Además, si tuviera que sustraer los efectos de la inducción pensando que de eso se trata un buen sondeo, cometería un error de sondeo. Terminará midiendo un voltaje que no corresponde a la configuración real del campo eléctrico en el circuito sino que corresponde solo a una parte del mismo (esto se muestra en la parte 5 donde el campo inducido se resta del campo total).

¿De qué se trata el "5 + 3 = 8 no es 5 + 3 - 8 = 0"?

Si hubiéramos optado por pasar de un terminal del inductor al otro siguiendo al conductor, habríamos tenido que abandonar KVL y recurrir a la ley de Faraday más general, porque la nueva ruta general del circuito estaría encerrando un campo magnético variable:

5+3=8

Y esta es la forma correcta de tratar los problemas con el cambio de flujo magnético: 5 + 3 = 8. No pretende que el voltaje sea independiente de la ruta (no lo es) y hace explícita la contribución del flujo magnético en la mano derecha lado de la ecuación.

Pero si estamos desesperados por volver a ver a nuestro amado KVL, podemos llevar el término del lado derecho al lado izquierdo, y terminamos con 5 + 3 - 8 = 0, la 'versión modificada' de KVL, donde el simulacro El voltaje del inductor ascendente es la derivada temporal del flujo magnético.
Esto es conveniente, porque podemos pretender que KVL todavía funciona, pero también es engañoso porque hace parecer que el voltaje es independiente de la ruta cuando en realidad no lo es.

4 - El anillo Romer-Lewin

En el anillo de Romer-Lewin, se colocan dos resistencias (agrupadas) dentro de una bobina que encierra una región de flujo variable, de modo que la ruta del circuito encierra una región dB/dt. Feynman es inflexible: no se puede utilizar la teoría de circuitos concentrados. Lewin también es inflexible: no se puede usar la ley de Kirchhoff (¡es para los pájaros!), por lo que debemos volver a la relación más general: la ley de Faraday.

Si acepta que el campo B variable generará un campo E rotacional, las cosas se pueden explicar observando cómo reaccionan las cargas en el circuito al campo inducido, cambian su configuración y producen un campo eléctrico resultante diferente dentro y cerca del anillo.

Si hiciera aparecer mágicamente el anillo hecho de cobre conductor y dos resistencias dentro de este campo circulante, las cargas dentro y en la superficie de sus partes se redistribuirían casi instantáneamente de tal manera que obedecieran la ley de Ohm.

Podemos echar un vistazo a lo que impulsa la redistribución de carga al considerar la ecuación de continuidad. Desplazamiento de carga: gradientes en la densidad de carga ρ - Ocurre en la superficie también en la dirección longitudinal, dondequiera que haya gradientes en los valores de conductividad y permeabilidad. si asumimos ρ t = 0 , entonces la ecuación de continuidad se convierte en div j = 0 y al sumar el cumplimiento de la ley de Ohm obtenemos

f1

si σ depende de las coordenadas, podemos expandir lo anterior como

f2

La densidad de carga está ligada al desplazamiento eléctrico, y en un medio de relativa permeabilidad ϵ r podemos expresar esto en términos de campo eléctrico por

f3

Eliminemos div E = -1/sigma . graduado sigma

f4

Y ahora expresemos E en términos de j = σ mi . Encontramos que la densidad de carga en el circuito es como

f5

y vemos que la densidad de carga, para una densidad de corriente dada, cambia según los gradientes de conductividad y permeabilidad.

Buena suerte en la solución de eso.

Pero cuando lo haces, ves que la carga se desplazará produciendo un campo eléctrico adicional que cancelará casi por completo la parte tangencial del campo eléctrico en el conductor. En un conductor perfecto con conductividad infinita, el campo tangencial sería cero en su interior. Con un valor finito de sigma, obtenemos un campo E tangencial resultante, compatible con la ecuación constitutiva j = σ mi . (Campo pequeño en cobre, campo grande en materiales resistivos).

Las cargas se distribuirán alrededor del conductor para producir un campo eléctrico muy pequeño, generalmente insignificante, dirigido axialmente a lo largo del conductor de cobre, y también se acumularán en la discontinuidad de la conductividad en los extremos de las resistencias, produciendo básicamente un voltaje coherente. con la ley de Ohm para la corriente resultante.

Resumiendo: si consideramos que la densidad de corriente j es la misma a lo largo de todo el circuito (tomando el conductor y la resistencia del mismo diámetro, solo que difieren en el material), veremos un campo eléctrico muy pequeño dentro del cobre altamente conductor ( por lo tanto, no hay distribución de voltajes de bobinas parciales imaginarias dentro del conductor), un gran campo eléctrico dentro de la pequeña resistencia (responsable del 'voltaje' de 0.1V a lo largo del camino que lo atraviesa, el signo depende de la dirección) y un eléctrico mucho más grande campo dentro de la resistencia más grande (responsable del 'voltaje' de 0.9V a lo largo del camino que lo atraviesa). La suma de todos los voltajes a lo largo del bucle (tenga en cuenta que tienen valores definidos porque estoy especificando una ruta) no es cero , pero es 1V, como lo predice la ley de Faraday.

Anillo de Romer-Lewin con campos y voltajes a lo largo del circuito

Tenga en cuenta que no hay sondas en el dibujo de arriba. Todos los voltajes que se muestran tienen un solo valor porque los estamos calculando especificando una ruta a lo largo del circuito (nuevamente, tenemos que especificar la ruta porque en campos no conservativos tales integrales dependen de la ruta). El hecho de que encontremos dos valores diferentes de voltaje entre A y B (es decir, 0,9 V y -0,1 V) es una consecuencia esperada del hecho de que toda la ruta del circuito encierra una región de campo magnético variable y, por lo tanto, la teoría del circuito ordinario es no aplica.

Entonces, la respuesta a su pregunta "¿cuál es el voltaje teórico entre los puntos A y B?" es " depende , ¿a lo largo de qué ruta quieres que se calcule?".

elige sabiamente tu camino, joven aprendiz

(Tuve que eliminar el resto, post demasiado largo)

Los valores -0,1 V y 0,9 V son caídas de voltaje en las resistencias, pero tiene que haber un voltaje inducido dentro de ese bucle que alimenta la corriente de 1 mA. Y no veo por qué el voltaje entre D y A no se puede definir en un momento dado. Entonces, según su teoría, el voltaje en una vuelta del devanado de un transformador tampoco se puede definir. Esta pregunta no se trata de KVL, se trata de la diferencia de potencial teórica entre los puntos D y A. Enfatice cuál debería ser el valor. Sería mejor una respuesta mucho más corta y específica, hay demasiada digresión en su respuesta.
@SlavenTojic La respuesta a su pregunta está en las dos primeras líneas: "Dado que la ruta de su circuito encierra una región de flujo magnético variable, la 'diferencia de potencial teórica' entre el punto D y A no está (únicamente) definida". Eso es todo al respecto. Entonces, no, no hay un valor "a" (incluso si se puede idear "un valor especial entre los muchos", en la parte sobre el vector potencial). Si cree que hay un 'voltaje distribuido' a lo largo del bucle, está equivocado: incluso en un autotransformador, donde puede 'extraer' el voltaje en cualquier bucle, el campo eléctrico dentro del cobre es cero.
Entonces, ¿por qué hay una corriente que fluye? De donde viene la corriente de 1mA
@SredniVashtar Gracias por responder esto. Me duele el cerebro intentar seguirlo, pero vota por el esfuerzo. Sin embargo, estoy con Slaven Tojic en esto, así que en lugar de decir que no hay un valor que se pueda idear para el voltaje entre D y A, tal vez pueda decir cuál es el voltaje entre D y A cuando la medición del voltímetro no está sujeta a inducción - como si estuviera colocada verticalmente sobre el experimento y en el centro del flujo de líneas de campo divergentes para que la inducción no pueda ocurrir.
@Andyaka sí, lo sé, es difícil dejar de lado la singularidad del potencial. Pero si lo piensas bien, esto es lo que sucede en cada bobina: si miras los terminales desde afuera, la integral de trayectoria es distinta de cero, pero si pasas por el conductor es cero. Simplemente nos engañamos a nosotros mismos de que solo hay un voltaje al no mirar dentro. En cuanto a mi respuesta, estoy de acuerdo en que duele el cerebro al leerla: es una mezcla de notas que hice en momentos separados, con diferentes convenciones, está llena de repeticiones y he detectado al menos dos errores de signo (arreglo en camino). Pero la esencia está ahí: no hay un voltaje único.
@SlavenTojic, ¿le sorprende que pueda tener una corriente que fluye en un conductor perfecto que tiene cero voltaje en sus extremos? Esto es lo que sucede literalmente en todos los circuitos. La corriente en el anillo de Romer-Lewin proviene de la inducción: el campo magnético cambiante provoca el campo eléctrico giratorio que provoca el desplazamiento de la carga que da como resultado un equilibrio donde hay una corriente de 1 mA que fluye a través del anillo y diferentes voltajes (calculados a lo largo del camino dentro de las resistencias) a través de las resistencias. No hay caídas de tensión en el conductor perfecto que los une. Bastante loco, ¿eh? Pero esa es la forma en que es
@SredniVashtar Su respuesta es demasiado larga, con muchas digresiones. Si realmente comprende este problema, puede explicarlo de una manera más breve y sencilla. Las preguntas principales son: ¿Por qué no puede modelar la inductancia de bucle como un inductor y colocarlo en el circuito? ¿Por qué el profesor está midiendo dos valores diferentes? Las soluciones mías y de Andy, también conocidas como, explican por qué está midiendo esos valores. Estaré encantado de aprender algo nuevo. Es bueno tener la mente abierta a nuevas ideas. Publiqué esta pregunta porque no era lógico que dos voltímetros midieran valores diferentes
@SlavenTojic La respuesta, como dije antes, está en las dos primeras líneas. Realmente no hay nada más. Las digresiones, estoy de acuerdo, son largas y aburridas, pero IMBO son necesarias para aclarar las muchas objeciones que generalmente surgen cuando esas dos líneas necesitan ser digeridas. También tuve una digresión sobre el transformador y cómo puede tocarlo en diferentes bobinas para obtener un voltaje de salida diferente cuando el campo eléctrico total es básicamente cero, pero no lo puse. Y se le ocurrió esa pregunta: - ). Las cosas pueden aclararse cuando llegue al vector potencial y la maniobra de McDonald :)
@SredniVashtar tiene que haber una explicación simple de por qué no se define el voltaje. No se puede decir simplemente que no está definido, es la forma en que es. Para citar a Albert Einstein: Si no puedes explicarlo de manera simple, no lo entiendes lo suficientemente bien.
Cuando los estudiantes no lo entienden, necesitan largas explicaciones.
@SlavenTojic Agregué una última imagen que debería abordar sus dudas sobre cómo un transformador puede producir un voltaje en sus terminales (abiertos, en este caso), cuando no hay acumulación de voltaje dentro del cobre de la bobina. Es posible que desee leer Purcell (Berkeley), pero también Haus & Melcher (MIT) o tratar de encontrar las lecciones de Arias (Cornell) en YT. Purcell y Haus también presentan el mismo experimento realizado por Lewin y están de acuerdo en que la interpretación correcta es que el voltaje no está definido de manera única. (No unicidad de voltaje en un sistema MQS: web.mit.edu/6.013_book/www/chapter10/10.0.html )
@SlavenTojic Ok, arreglé (al menos el origen y el final de) las líneas de campo. No son simulados, solo esbozados sobre la marcha. Si desea ver las líneas de campo reales en la parte interna de una bobina, puede encontrarlas en el libro de texto de Haus y Melcher mencionado anteriormente. (una nota final: el transformador ideal es una idealización que tiene una autoinductancia infinita; no es el caso del bucle Romer-Lewin de una sola vuelta. Lo que modelo aquí es el transformador real (si consideramos que la bobina primaria produce el flujo), sin pérdidas y con una autoinducción despreciable porque eso es lo que utiliza Lewin).
Si realmente simula el campo, encontrará que en el caso electrostático si calcula la integral de trayectoria a lo largo de una trayectoria de un punto a otro de la superficie del conductor en el espacio que lo rodea, se encontrará yendo en contra del campo E en el primer parte, a lo largo del campo en el medio y contra el campo en la última parte. El resultado es una integral de trayectoria cero, como se esperaba en el caso electrostático. En el caso de MagnetoQuasistatic, en cambio, los caminos de una terminal a otra fuera del conductor tendrán una parte dominante que va a lo largo o en contra del campo dando un voltaje distinto de cero.
Muchas gracias por la respuesta, se me hace bastante pesada. Intentaré revisar todo para mañana
Por cierto, sugiero completar esta publicación @SredniVashtar. Esto tiene el potencial de ayudar a muchas personas a largo plazo/brindar mucha información a otros.
@Buraian, lamentablemente, no hay más espacio para agregar (tuve que cortar parte de esta publicación para mantenerme en el límite de 30k caracteres). Planeé agregar dos respuestas más a una pregunta similar sobre el anillo de Lewin: una usando mallas y otra usando campos, pero seguí posponiéndola porque era tiempo. Una oración a los moderadores: no eliminen los comentarios técnicos en la sección de comentarios de esta publicación, porque sirven como una extensión de las explicaciones que no pude agregar debido al límite de longitud alcanzado.
@SredniVashtar ¿Tiene un plan para completar su respuesta? Si la publicación es demasiado larga, no es raro publicar varias respuestas en Stack Exchange. Estoy especialmente interesado en su explicación de la inductancia parcial, ya que su respuesta me hizo darme cuenta de que los conceptos de inductancias concentradas y parciales a menudo se abusan todo el tiempo en la ingeniería de RF/EMC. Hablamos de "la inductancia parásita de un cable/un componente" y pensamos en ella en términos de teoría de circuitos concentrados. Es una guía útil para obtener una descripción cualitativa o una cifra aproximada, pero es físicamente incorrecta.

Alteraciones significativas por error básico

El circuito real en la pregunta es este: -

ingrese la descripción de la imagen aquí

Letra pequeña añadida el 27 de junio de 2020

  • Lewin afirma que el flujo aumenta linealmente a una velocidad que induce 1 voltio en el bucle (lo llamo el "bucle principal").

  • También asumo que todas las líneas de campo de retorno que fluyen hacia la bobina productora de flujo están fuera del bucle formado por R1, el punto D, R2 y el punto A.

  • También asumo que los dispositivos de medición M1 y M2 (que se muestran a continuación) no afectan ni se ven afectados por los campos magnéticos.

  • También asumo que el cableado de medición M1 y M2 (a los puntos D y A) toman una ruta que está muy cerca de la ruta del bucle principal, es decir, reciben el mismo nivel de inducción que el bucle principal.


En otras palabras, con 1 mA CC fluyendo alrededor del bucle debido a un campo magnético que aumenta linealmente, el voltaje en R1 debe ser de -0,1 voltios y el voltaje en R2 debe ser de +0,9 voltios. Esto no puede ser discutido. He agregado signos a los valores ahora para aclarar esto.

La diferencia de voltaje que se ve entre las dos resistencias se debe a que cada punto en el bucle tiene inductancia (incluidas las resistencias) y esta es una inductancia distribuida , por lo tanto, el voltaje inducido también se distribuye y esto significa que hay una caída de voltaje en los cables que conectan la parte superior de R1 con la parte superior de R2 y lo mismo con el nodo inferior. Eso explica la diferencia de voltaje: -

ingrese la descripción de la imagen aquí

  • L1, L2, L3 y L4 representan la inductancia que recibe voltaje inducido y la he dividido en cuatro cantidades para facilitar la comprensión.
  • Si midieses (con cuidado) directamente entre los puntos D y A medirías 0,4 voltios (anteriormente dije 0,5 voltios pero eso era un error estúpido ).
  • Si da la vuelta al bucle mirando los voltajes inducidos (en azul), verá que el voltaje inducido total es de 1 voltio (4 x 0,25 voltios).
  • Ese 1 voltio inducido impulsa 1 mA a través de la resistencia total de 1000 ohmios. (La corriente se muestra con flechas rosas)
  • El voltaje a través de cada resistencia es como se indica porque V = I R y podemos suponer que la longitud física de cada resistencia es tan pequeña que podemos ignorar algunos milivoltios inducidos a través de cada resistencia que disminuirían ligeramente sus caídas de voltios IR locales.

Debido a que el experimento de video no está definido de manera adecuada, es difícil estar seguro, pero suponiendo que los nodos de medición estén exactamente en los puntos D y A (en lugar de directamente a través de cada resistencia), entonces el bucle de medición que sirve a R1 reflejará los -0.1 voltios vistos. porque los lazos de medición son tan vulnerables a la inducción como el lazo de corriente principal.

Es lo mismo para el bucle de medición que sirve a R2: -

ingrese la descripción de la imagen aquí

  • Entonces, M1 leerá 0,4 voltios menos el doble de 0,25 voltios = -0,1 voltios
  • Y, M2 leerá 0,4 voltios más el doble de 0,25 voltios = +0,9 voltios

Si no puede ver esto fácilmente, piense en el terminal + de M1. Tiene que ser 0,25 voltios más bajo que el punto D, mientras que para M2, su terminal + tiene que ser 0,25 voltios más alto que el punto D. Una historia similar para los terminales del medidor en relación con el punto A.


Y, en el caso de que las dos resistencias (ahora llamadas R3 y R4) tengan valores iguales de 500 ohmios, si pudiera medir directamente entre D y A, creo que mediría 0 voltios: -

ingrese la descripción de la imagen aquí

  • M1 mediría -0,5 voltios y
  • M2 mediría +0,5 voltios

Antes de las modificaciones de hoy, me estaba confundiendo con esto, pero ahora está más claro. No ayudó que cometí el error de calcular el voltaje incorrecto entre D y A en el primer ejemplo; eso me desconcertó y me llevó un par de días asimilar y ver lo obvio. Memo para mí mismo: ¡siempre revisa las cosas dos veces!

@Andyaka Me sorprende que hayas intentado aplicar las reglas de Kirchhoff donde no pertenecen. Debería volver a pensar en este problema y en el concepto de inductancias 'parciales' o 'distribuidas'. Eche un vistazo a cómo trataron este problema en el intercambio de pila de física, o a los comentarios menos votados en el canal de videos de youtube de Electroboom (pruebe los de Trevor Kearney, los repite a menudo, por lo que son más fáciles de encontrar). El potencial simplemente no está definido de manera única.
@SredniVashtar, ¿por qué la fijación con Kirchoff? No pensé en Kirtchoff en absoluto cuando resolví esto. Si cree que está mal y el SE de física lo muestra, por favor enlace. Estaré encantado de echarle un vistazo. Por cierto, no estoy en desacuerdo con las observaciones eléctricas de Lewin.
@SredniVashtar ¿Te refieres a esta pregunta ?
Los comentarios no son para una discusión extensa; esta conversación se ha movido a chat .
@VoltageSpike si elimina esos comentarios antes de incluir el que tengo texto en rojo, entonces será un mejor movimiento.
Usaste KVL para hacer que los voltajes alrededor del bucle fueran cero, ¿no? La teoría del circuito de agrupamiento no funcionará aquí porque la región cambiante del flujo magnético es parte del circuito. Feynman es muy firme al respecto. Agregar inductancias agrupadas a lo largo del camino es como explicar las órbitas de los planetas con epiciclos. Podría darle los números que desea, pero no es la física correcta. Según su circuito, parece que hay 0.25 V en una pieza de alambre de cobre altamente conductor. Y, sin embargo, sabemos que el campo eléctrico dentro del cable es casi cero (todo el campo relevante está dentro de las resistencias).
@SredniVashtar, una pieza de cobre con una resistencia de casi cero ohmios no evita que produzca un voltaje terminal cuando hay un campo magnético cambiante. Después de todo, Lewin no tuvo problemas para elogiar a Faraday en esa conferencia. De hecho, nunca uso KVL a sabiendas porque lo encuentro demasiado poco intuitivo. Sumar un montón de voltajes en serie es como medir los lados de un cuadrado y totalizarlos. De todos modos, proporcione un enlace a donde sea si solo va a decir cosas como esa. ¿De verdad está diciendo que los números son incorrectos? Si es así, por favor enlace a lo que deberían ser.
Está utilizando un circuito agrupado, que por definición requiere que todo el flujo magnético cambiante esté encerrado exclusivamente dentro de los componentes y no esté encerrado por la ruta del circuito para modelar un circuito cuya ruta encierra un flujo magnético cambiante (Feynman dice que no puede hacer eso: feynmanlectures.caltech.edu/II_22.html ). En cuanto al inductor que tiene un voltaje en los terminales, bueno, los terminales están fuera de la región de flujo magnético cambiante, y esa es la clave (Ramo Whinnery VanDuzer explica esto muy bien en "Campos y ondas en la electrónica de comunicación", cap. 4).
@SredniVashtar bien, por favor responda. Es posible que haya dibujado inductores agrupados, pero solo los usé para explicar la inducción de 1 voltio. También insistí en que las resistencias eran muy cortas y, por lo tanto, se podía descontar el voltaje inducido entre ellas. Si los números son incorrectos, explíquelo.
Los números de las resistencias no son incorrectos y, de hecho, reflejan el hecho de que el campo E no es conservativo y alrededor del bucle la integral de línea no es cero. Lo que está mal es tratar de agrupar inductancias a lo largo del camino para hacer que KVL funcione y hacer que la línea sea integral cero (cuando no lo es). Nuevamente, el problema está en la ruta del circuito que encierra el campo cambiante, no en los componentes individuales. Este i.ibb.co/270w7VP/screenshot-3.png muestra el campo eléctrico total: no hay casi nada a lo largo del camino; todo sucede en la resistencia y si completa el círculo, obtiene un valor distinto de cero.
@SredniVashtar según su teoría, el voltaje inducido en un bucle de cable depende de la resistencia del cable, pero la ley de Faraday establece que el voltaje inducido depende de la inductancia del cable: fem = L * di / dt
@SlavenTojic No. La resistencia de la que estaba hablando es la agrupada en las resistencias de 100 y 900 ohmios: cuando el circuito se coloca en el campo de rotación, desarrollan cargas superficiales que crean el campo en su interior). La bobina tiene idealmente resistencia cero. Si lo encuentra, eche un vistazo a Ramo Whinnery VanDuzer, capítulo 4 citado anteriormente. (si tengo tiempo, produciré una respuesta, con una mención al papel del vector potencial, pero necesito encontrar todos mis dibujos, no quiero recrearlos desde cero)
SredniVashtar Tomemos este ejemplo, la mitad izquierda del bucle consta de un cable cuya resistencia es de 100 ohmios, la mitad derecha del bucle consta de un cable que tiene 900 ohmios. Los materiales de cada mitad del bucle son diferentes. Ahora, si toma la ley de Faraday, la mitad izquierda y derecha del bucle tienen la misma cantidad de voltaje inducido porque emf = d (B * S) / dt
@SredniVashtar responda, por favor, porque su diagrama no fue útil porque no consideró la inducción alrededor del ciclo: hablar de la densidad de corriente dividida por la conductividad parece perder el punto sobre la inducción. Claramente, cualquiera que responda considerará que el cobre es un conductor perfecto, pero eso no significa que no sea susceptible a la inducción de un campo magnético que aumenta linealmente (o cualquier campo magnético cambiante).
Voy a dar una respuesta, pero va a tomar un tiempo. Mientras tanto: tomé en cuenta la inducción, si recorres el bucle obtienes 1V, eso es inducido por el campo magnético cambiante. En cuanto al problema cambiado con los medios anillos hechos de diferentes materiales: esto complica las cosas porque las cargas superficiales interactúan entre sí, pero creo recordar que Lewin también consideró este caso en uno de sus videos. Básicamente, la densidad de carga depende de los gradientes de conductividad y permeabilidad y tenemos cambios abruptos de ambos en el límite entre los semi-anillos y en su superficie lateral.
@SlavenTojic: ¿has cambiado de opinión? ¿Ha descubierto algo sobre el experimento que hace que una respuesta diferente sea más apropiada?
@Andyaka Lo siento, no creo que su respuesta sea incorrecta, pero tampoco estoy seguro de que la respuesta de SredniVashtar sea incorrecta, por lo que eliminé la respuesta aceptada. Este problema requiere un enfoque experimental. Lo que me molesta es cómo se puede inducir una diferencia de potencial en un circuito cerrado, en un circuito abierto las cargas eléctricas se empujan hacia los extremos del circuito abierto, pero en un circuito cerrado no tiene sentido. ¿En qué parte del bucle ocurriría la diferencia de potencial?
Recuerde que el flujo aumenta linealmente y que el bucle tiene inductancia, por lo tanto
V = L d i d t
En otras palabras, no debe considerar el bucle como corto pero con inductancia. De todos modos, esa es la forma en que lo veo.
@Andyaka Al principio lo veo igual que tú, pero no estoy 100% seguro. No aceptaré ninguna respuesta hasta que esté 100% seguro de cuál es la respuesta correcta.
No hay problema amigo.
@SlavenTojic en el anillo de Lewin con resistencias, la carga que ha sido desplazada por el campo rotacional inducido se acumulará en los límites de la resistencia, dando lugar al fuerte campo eléctrico dentro de ellos y, por superposición (entre los campos inducidos y colombianos generados por la interfaz y cargas superficiales), al pequeño campo E = j / sigma resultante dirigido a lo largo del conductor en el verso de la corriente. En un anillo resistivo uniforme, debe imaginar las resistencias distribuidas a lo largo del anillo de modo que el único campo resultante sea E = j / sigma.
Además, la explicación L di/dt no funciona. En primer lugar, en el experimento de Lewin la autoinductancia L es despreciable, y ni siquiera M di/dt puede salvarte porque bueno, puedo hacer el mismo experimento con un imán que cae. Desafortunadamente, llegué al límite de 30k caracteres y no puedo agregar el resto de mi respuesta. Podría escribir una segunda respuesta a otra pregunta relacionada con la paradoja de Lewin en este sitio, veré si encuentro el tiempo (la parte sobre la inductancia parcial es bastante interesante...)

Al calcular un cambio de corriente debido a un cambio de flujo magnético, ¿qué puede ser más "ideal" que un bucle circular de resistencia cero en un campo magnético uniforme con la fuerza del campo aumentando/disminuyendo linealmente en el tiempo? Con la ayuda de la ecuación de Maxwell-Faraday en formulación integral (ley de Faraday) llegamos inmediatamente a la respuesta.

Pero a menudo se pasa por alto que el camino a la solución mediante la aplicación de la ley de voltaje de Kirchhoff no es más difícil. Primero, tenemos un EMF generado por un campo magnético externo (un valor dado EMF_ext). Luego, en un cable de resistencia cero, la ley de Ohm prohíbe cualquier caída de voltaje distinta de cero, por lo que un EMF de un campo magnético generado por una corriente en el bucle (un valor) tiene que compensar por EMF_currentcompleto E_ext. EMF_current + EMF_ext = 0, y los cálculos adicionales son triviales. Una cosa a tener en cuenta: la formulación exacta de la ley de voltaje de Kirchhoff establece

La segunda ley de Kirchhoff es la siguiente: la suma de las fuerzas electromotrices en un bucle es igual a la suma de las caídas de potencial en el bucle.

Lo que lleva a un malentendido mutuo (como en nuestra discusión) es una omisión de la contribución de la fem o, más precisamente, la sustitución temprana de algunas caídas de voltaje inventadas por la contribución de la fem en el análisis del circuito de alguien . La parte fem de la formulación exacta de Kirchhoff falta incluso en un artículo de Wikipedia sobre las leyes del circuito de Kirchhoff.

El concepto de EMF a menudo se atribuye a la descripción de acciones de fuentes de voltaje no eléctricas como baterías químicas, termopares, componentes fotovoltaicos, etc. en redes. Además, a menudo se argumenta que Kirchhoff no es válido para aplicaciones fuera de los modelos de parámetros agrupados. Argumento que las leyes de Kirchhoff se cumplen en todas las aplicaciones, incluso cuando se consideran los fenómenos de radiación EM. Un ejemplo interesante de "reconciliación" de las leyes de Kirchhoff y los cálculos de diseño de antena se puede ver aquí y en las referencias dadas en este artículo.

Claro, utilizará algún solucionador FEM EM para el diseño avanzado de antenas. Pero no hay nada contradictorio en aplicar Kirchhoff a cualquier parte del circuito de su antena, incluidos los extremos de una antena dipolo.

En conclusión, aprovecho la oportunidad para anotar un EMF generado al cambiar el flujo magnético debido al cambio de corriente.

mi METRO F C tu r r mi norte t = L d I / d t

Observe el signo menos en esta expresión.

Usted dice que "un EMF de un campo magnético generado por una corriente en el bucle (un valor EMF_current) tiene que compensar completamente E_ext". Y das - L di/dt por eso. El problema es que la contribución debida a la autoinducción del anillo de Romer-Lewin es insignificante (en los casos de excitación "primaria" de pulso, rampa y sinusoidal empleados por Lewin, el experimento mental y la configuración de Mabilde). ¿Cuidado para elaborar?
Sí, me importa. ¿Está solicitando una extensión del análisis práctico teórico EM/EE a un caso de bucle de resistencia distinto de cero?
No, estoy preguntando cómo se puede compensar una fem de 1 V con menos de mV L di / dt (cero, en el caso de corriente de bucle constante).
1V emf no se puede compensar con una cantidad cero, y nadie afirma lo contrario. Un participante en esta discusión tiene razón en que el problema tiene una explicación más simple que las explicaciones que los demás intentaron ofrecer. Acabo de dar una pista para un enfoque sólido para el análisis de inductores en campos magnéticos cambiantes. ¿Te importaría hacer una pregunta y leer una explicación?
No, estoy preguntando cómo se puede compensar una fem de 1 V con menos de mV L di / dt (cero, en el caso de corriente de bucle constante). Para obtener más detalles, consulte youtube.com/watch?v=wz_GqO-Urk4 a las 12:24 y siguientes. En la configuración de Lewin, la contribución de autoinducción está en el rango de microvoltios. EDITAR: escribió "entonces, un EMF de un campo magnético generado por una corriente en el bucle (un valor EMF_current) tiene que compensar completamente E_ext. EMF_current + EMF_ext = 0" (Iba a cancelar mi comentario anterior, pero lo haré dejalo)
1) La fem de 1 V no puede compensarse con ninguna cantidad que no sea igual a 1 V. ¿Por qué miras clips para asegurarte de que es verdad? 2) La medición de autoinducción de alguien resultó en una lectura de unos pocos uV. Se han realizado enormes mediciones de voltajes; 3) En mi análisis de una configuración de bucle de resistencia cero, EMF_ext + EMF_current = 0, y afirmo que mi análisis puede ayudarlo a comprender mejor el comportamiento de los circuitos con inductor en campos magnéticos cambiantes. Entonces, ¿qué preguntas? ¿Quieres que te ayude a dar el siguiente paso? ¿Para extender un análisis en una red con resistencias? ¿O que?
Ok, probemos de esta manera: ¿EMF_ext = 1V? ¿Es EMF_curr=-1V? Si EMF_curr = -1V y L = 1 microhenry, ¿di/dt no es cero? Y si el anillo está hecho de un material resistivo uniforme, ¿di/dt sigue siendo diferente de cero? (aquí i es la corriente en el segundo bucle, no el que genera el campo magnético). Y en el anillo de Romer-Lewin (con una corriente que aumenta de forma lineal e indefinida en el primer bucle), ¿di/dt todavía no es cero? Es posible que desee agregar esta información a su respuesta.
DE ACUERDO. No editaré mi respuesta aquí; esta respuesta cumple su propósito perfectamente. Puedes hacer una nueva pregunta. Por ejemplo, puede preguntar sobre una forma de calcular una corriente en un bucle conductor en un campo magnético cambiante. Para advertir respuestas fuera de tema, puede indicar que puede hacer estos cálculos con la ley de Faraday y J = sigma * E, pero le gusta leer cómo se puede hacer con fem inducida, Kirchhof y Ohm. También puede pedir que se extienda sobre posibles análisis de potenciales en ambas formas de razonamiento. Si hace la pregunta, proporcione una referencia en un comentario a esta pregunta.
Y por favor, no necesites ver clips. Coloque la información pertinente con la pregunta. También puede hacer que la pregunta sea un desafío: invitar solo respuestas limitadas por un número específico de palabras
Primero, se agregó el video como referencia. Toda la información relevante estaba en el comentario, es decir, L y di/dt en el circuito de la pregunta no brindan ninguna contribución apreciable. Esto hace que su respuesta a la pregunta del OP sea OT, ya que esa información no se aplica al circuito en la pregunta. En segundo lugar, ¿por qué debería hacer una pregunta sobre un método que a mi modo de ver es incorrecto? O hace su propia pregunta y la responde, o agrega una segunda respuesta a esta pregunta, abordando el circuito con el que tiene problemas el OP.
Señor Sredni Vashtar, gracias por no rechazar mi respuesta. Tiene razón, formalmente no es una respuesta a la pregunta de OP, solo una pista para usted y los demás participantes que indica la fuente de su malentendido mutuo. Espero que, a su debido tiempo, domine un enfoque EE para el análisis general de circuitos (si es lo que usted llama "un método que, en mi opinión, es incorrecto") y lo aplique con éxito para resolver los problemas apropiados. Le aseguro que el problema que discutió en su respuesta a la pregunta de OP es susceptible de un enfoque puramente emf-Kirchgoff's-Ohm. Buena suerte.
Ok, tenemos que creer en su palabra, como es costumbre en la comunidad científica: escriba el resumen y deséele buena suerte. ¿Esta rama de la física se llama Shy Phy? :-) es una broma. En cuanto a la resolución, puedo hacerlo hasta en un comentario en una sola ecuación: escribir la circulación de E (R1 i + R2 i) y equipararla a -dphi/dt (emf, o 1V). Eso es todo: obtenemos i, y luego podemos encontrar los voltajes en las dos resistencias. Sí, son diferentes, como era de esperar. Es una consecuencia del cálculo básico y la física básica. Explicar esto a los Kirchoffianos requeriría otras dos respuestas de 30k caracteres, aunque.
Los "kirchhoffianos" no me piden que les explique cómo resolver el problema del inductor en una aproximación cuasiestática con un modelo de parámetros concentrados, lo hacen. Haga una pregunta SE y le prometo que responderá con mucho menos de 30 000 caracteres de texto. Luego, podrá decidir por sí mismo si la aplicación del método "kirchhoffiano" a problemas cuasiestáticos es teóricamente impecable y prácticamente eficiente en la realidad o no, y compartir sus conclusiones.
@SredniVashtar Tiene razón en que el campo dentro de un conductor perfecto es cero, por lo tanto, la caída de voltaje en el conductor es cero. El problema es que no está teniendo en cuenta la inductancia del bucle, que también es una propiedad del conductor. En la inductancia del bucle está el voltaje inducido e=L di/dt, los voltajes a través de los resistores son caídas de voltaje y no son voltajes inducidos, porque las caídas de voltaje son causadas por una corriente que pasa por un resistor y el voltaje inducido en un El bucle es causado por un flujo magnético cambiante a través de ese bucle.
@SlavenTojic primero: ¿crees que los electrones van con un contador que realiza un seguimiento de quién donó el campo eléctrico a quién? Simplemente experimentan el campo eléctrico total. Y el campo que da lugar a una caída de voltaje cero dentro de la bobina y el voltaje distinto de cero en los terminales (ya sea debido a autoinducción, autoinductancia, imán que cae o explosión nuclear) es la suma del campo de inducción rotacional (que desplaza los cargos) con el campo colombino del cargo que ha sido desplazado. ¿Por qué quieres contar el campo de inducción dos veces? Ya se ha 'gastado'.
segundo: no importa si el voltaje se debe a la inductancia propia o mutua o a un imán que cae; el campo E total en el interior es cero, pero ¿por qué siguen mencionando la autoinductancia? Su contribución es irrelevante: en el experimento donde el campo aumenta constantemente, la corriente en el bucle es constante, por lo tanto, di/dt = 0 y también lo es L di/dt. aporte CERO. En el experimento de Lewin con corrientes pulsadas, ¡¡¡L di/dt representa un microvoltio!!! ¿Por qué molestarse en hacer un seguimiento de eso? Antes de abordar el anillo de Romer-Lewin, le sugiero que estudie cómo funciona un inductor. No hay acumulación de voltaje en el interior.
@SredniVashtar Según usted, la teoría del transformador ideal también es para las aves. De acuerdo con su teoría, el voltaje inducido dentro de los devanados del transformador es cero, porque no hay resistencia dentro de los devanados del transformador, por lo tanto, el campo es cero y no se induce voltaje. Hay una diferencia entre la caída de voltaje y el voltaje inducido. El voltaje inducido hace que la corriente fluya y la corriente provoca caídas de voltaje en las resistencias.
@SlavenTojic Sí, dentro de los devanados hay un campo cero y, por lo tanto, un voltaje cero. Todo el voltaje aparece en los terminales (Ramo Whinnery Vanduzer lo confirma). Los electrones no pueden distinguir las componentes colombiana e inducida del campo, solo ven el campo total. Si descompone el campo, también descompondrá el voltaje en una parte conservativa que depende solo de los puntos finales y una parte no conservativa que depende de la ruta (Popovic muestra esto). El voltaje general dependerá de la ruta y será cero dentro de los devanados y distinto de cero afuera. Véase también Purcell o Haus.

En contraste con la respuesta impresionante y completa de @Sredni, la mía será bastante más corta.

Conecte un multímetro ordinario entre los dos puntos de medición. Coloque el medidor en el lado derecho del experimento y coloque sus cables fuera del área de cambio de flujo. Indicará 0,9 V. Ahora coloque el medidor a la izquierda del experimento y coloque sus cables fuera del área de cambio de flujo. Leerá 0.1 V.

Piense en qué flujo encierran los cables en cada circunstancia. De hecho, conecte los cables del medidor, deseche las resistencias y los cables del experimento, y simplemente coloque el medidor y su bucle de cables en varios lugares en el área de cambio de flujo, y vea cómo cambia su lectura.

Hágalo como un experimento mental, o como un experimento real si puede hacer un área lo suficientemente grande para cambiar el flujo magnético. Puede reemplazar los cables del medidor por un pequeño lazo de alambre de cobre si desea reducir la escala. Puede reemplazar el bucle de alambre de cobre por muchas vueltas si desea aumentar la sensibilidad.